Solving Set Equality Proof Homework

  • Thread starter Thread starter Haydo
  • Start date Start date
  • Tags Tags
    Proof Set Sets
Click For Summary
The discussion revolves around proving the set equality (A-B)∪C = (A∪C)-(B∪C) given that A is a subset of B. The initial approach involves demonstrating that each side is a subset of the other, but confusion arises regarding the role of set C in the proof. It is noted that if an element x is in C, it cannot simultaneously satisfy the conditions for being in the right-hand side of the equation. A Venn diagram is suggested as a helpful tool to visualize the relationships between the sets. Ultimately, the conclusion is reached that the problem may contain an error, prompting the decision to contact the professor for clarification.
Haydo
Messages
20
Reaction score
0

Homework Statement



Let ##A, B, C## be sets with ##A \subseteq B##. Show ##(A-B)\cup C=(A\cup C)-(B\cup C)##

Homework Equations



None.

The Attempt at a Solution



So, generally, one shows two sets to be equal by showing that each is a proper subset of the other. I started with the LHS. Thus, if x is in (A-B)UC, x is in (A-B) or x is in C. But if x is in C, then x is not in RHS. So it seems that the expression does not hold. Am I thinking of this wrong, or did the assignment writer make an error?
 
Last edited by a moderator:
Physics news on Phys.org
If x is in C, then x can still be in the right hand side. It might help to draw a Venn diagram.
 
I should say, consider the case where x is in C and x is not in (A-B). Then x cannot be in (AUC)-(BUC), right?
 
You know what, I think you're right! If x is in C, then for x to be in the RHS it could not be in ## B \cup C ##. But then it cannot be in ## C ##. Also, my first Venn diagram had a mistake in it.
 
Hm. That's what I thought. I'll email the professor then. Maybe he was just trying to see if we were paying attention, but the problem definitely said to prove the equality, not to prove it or show it is false.
 
Question: A clock's minute hand has length 4 and its hour hand has length 3. What is the distance between the tips at the moment when it is increasing most rapidly?(Putnam Exam Question) Answer: Making assumption that both the hands moves at constant angular velocities, the answer is ## \sqrt{7} .## But don't you think this assumption is somewhat doubtful and wrong?

Similar threads

  • · Replies 3 ·
Replies
3
Views
1K
  • · Replies 6 ·
Replies
6
Views
2K
  • · Replies 8 ·
Replies
8
Views
2K
Replies
1
Views
2K
  • · Replies 6 ·
Replies
6
Views
4K
  • · Replies 2 ·
Replies
2
Views
2K
  • · Replies 9 ·
Replies
9
Views
2K
  • · Replies 1 ·
Replies
1
Views
2K
  • · Replies 3 ·
Replies
3
Views
2K
Replies
10
Views
2K